1
$\begingroup$

How to model the Linear Programming for the problem below with the most complete + reasonable constraints.
A production facility has 2 types of reinforcement bars 6m, 8m long (unlimited quantity). Need to process 100 2.4m and 150 2.8m sections. Ask how to cut rebar to save the most?
This is a practice homework in my school. It bother me for a while. I don't know how to set variables. I set number of 2.4m bar, 2.8m bar in 6m bar is x, y; 2.4m bar, 2.8m bar in 8m bar is a, b; then number of 6m, 8m bar is n1, n2. Then one of constraints is $n_1x+n_2a\leq100$. That is non-linear so I can't continue solving with integer programming. Can someone help? Thank you.

$\endgroup$

1 Answer 1

1
$\begingroup$

How can we cut the bars to 2.4 and 2.8 pieces? Let $x_i$ and $y_i$ be the different cuts as follows

$$\begin{aligned} x_1 &: 6 = 2.4+2.4+\color{red}{1.2} \\ x_2 &: 6 = 2.4+2.8+\color{red}{0.8} \\ x_3 &: 6 = 2.8+2.8+\color{red}{0.4} \\ y_1 &: 8 = 2.4+2.4+2.4+\color{red}{0.8} \\ y_2 &: 8 = 2.4+2.4+2.8+\color{red}{0.4} \\ y_3 &: 8 = 2.4+2.8+2.8 \\ \end{aligned}$$

So we should minimize the number of bars used.

\begin{align} \min \quad& z= x_1+x_2+x_3+y_1+y_2+y_3 \label{z1} \\ \text{s.t.} \quad& 2x_1+x_2+3y_1+2y_2+y_2 \ge 100 \tag{1}\label{c1} \\ & x_2+2x_3+y_2+2y_3 \ge 150 \tag{2}\label{c2} \\ & x_i, y_j \geq 0 \space \text{(int)}. \tag{3}\label{c3} \end{align}

$\endgroup$
2
  • $\begingroup$ I think $min\quad z=1.2x_1+0.8x_2+0.4x_3+0.8y_1+0.4y_2$ right? $\endgroup$ Commented Jun 29, 2022 at 6:53
  • $\begingroup$ @BùiTrườngGiang If you want minimize the number of bars used you should $min(\sum{x_i} +\sum{y_i})$ but if you want to minimize wasted pieces yes you rigth. $\endgroup$ Commented Jun 29, 2022 at 6:55

You must log in to answer this question.

Start asking to get answers

Find the answer to your question by asking.

Ask question

Explore related questions

See similar questions with these tags.